Nonmeasurable Set with Finite Outer Measure

Click For Summary
The discussion revolves around proving the existence of a Gδ set G that contains a nonmeasurable set E of finite outer measure, such that the outer measure of G equals that of E, while the measure of the difference G-E is greater than zero. It is noted that E being nonmeasurable complicates the direct application of existing theorems, but a strategy involving the axiom of choice is proposed. The approach suggests constructing measurable sets G_n that approximate E closely in measure and then finding a G that satisfies the required conditions. The necessity of the axiom of choice is debated, particularly in relation to constructing nonmeasurable sets like Vitali sets. Ultimately, the proof concludes that such a Gδ set G can indeed be formed under the given constraints.
Bashyboy
Messages
1,419
Reaction score
5

Homework Statement


Let ##E## be a nonmeasurable set of finite outer measure. Show that there is a ##G_\delta## set ##G## that contains ##E## for which ##m^*(E)=m^*(G)##, while ##m^*(G-E) > 0##.

Homework Equations



##E## is a measurable set if and only if there is a ##G_\delta## set ##G## containing ##E## for which ##m^*(G-E)=0##.

Let ##E## have finite outer measure. Show that there is an ##F_\sigma## set ##F## and a ##G_\delta## set ##G## such that ##F \subseteq E \subseteq G## and ##m^*(F)=m^*(E)=m^*(G)##.

The Attempt at a Solution



I pretty certain it wouldn't follow immediately from the two theorems I quoted in section 2, right? In fact, we could prove an analogous theorem for ##F_\sigma## sets, right?
 
Physics news on Phys.org
Since E is nonmeasurable, if we can find a measurable set G that contains it and has the same outer measure, the result will follow from the first theorem you quote in section 2.

Are you allowed to use the axiom of choice? If so, I suggest the following strategy.
  1. prove that, for any positive integer ##n##, there exists a measurable set ##G_n\supset E## such that ##m^*(G_n)-m^*(E)<1/n##.
  2. find a way to construct a measurable set ##G\supset E## such that ##m^*(G)-m^*(E)=0##.
Part 2 is easier than part 1. But I have a feeling that 1 is provable, given some key properties of outer measures and measurable sets.

If you're not allowed to use axiom of choice, I don't know whether it's still true. If it is, a more subtle proof may be needed, as the axiom is needed for the approach I had in mind for step 2. IIRC, the axiom is usually assumed in measure theory, as the construction of the most famous nonmeasurable sets, the Vitali sets, requires it.
 
I don't understand why the axiom of choice is needed. Here is proof I came up with:

Suppose that ##E## is a nonmeasurable set of finite outer measure. Then for every ##G_\delta## set ##G## containing ##E## is such that ##m^*(G-E) > 0## (negation of first theorem. Moreover, since ##E## has finite out measure, there exists a ##G_\delta## set ##G## such that ##m^*(G)=m^*(E)## (second theorem). Hence, we have found a ##G_\delta## set ##G## containing ##E## for which ##m^*(G)=m^*(E)## and ##m^*(G-E) > 0##.
 
Question: A clock's minute hand has length 4 and its hour hand has length 3. What is the distance between the tips at the moment when it is increasing most rapidly?(Putnam Exam Question) Answer: Making assumption that both the hands moves at constant angular velocities, the answer is ## \sqrt{7} .## But don't you think this assumption is somewhat doubtful and wrong?

Similar threads

Replies
1
Views
2K
  • · Replies 1 ·
Replies
1
Views
2K
  • · Replies 1 ·
Replies
1
Views
2K
Replies
1
Views
2K
  • · Replies 3 ·
Replies
3
Views
3K
Replies
2
Views
2K
Replies
12
Views
2K
Replies
2
Views
2K
  • · Replies 2 ·
Replies
2
Views
3K
  • · Replies 2 ·
Replies
2
Views
3K